Assignment 4
Exercise 1 Give an example of the following, or explain why no such example can exist.
- A sequence with infinitely many terms equal to zero, but does not converge to zero.
- A sequence with infinitely many terms equal to zero, which converges to a nonzero number.
- A sequence of irrational numbers that converges to a rational number.
- A convergent sequence where every term is an integer.
- Two divergent sequences where is convergent.
- Two divergent sequences where is convergent.
Solution.
This is possible: the sequence has every other term equal to zero but is divergent.
This is impossible. If and , then for we can find an for which all have . Unpacking this, Whether is greater than or less than zero, this interval does not contain zero. So, no terms past can possibly be zero, and there must only be finitely many terms equal to zero (they can only be in the terms )
This is possible: given any irrational the sequence converges to by the limit theorems.
This is possible: take the constant sequence
Let and . Then each sequence diverges, but is the constant sequence which converges to .
Let and . Each of these is divergent but their product is the zero sequence so this is convergent.
Exercise 2 Prove, directly from the definition of convergence (not using limit theorems / squeeze theorem / etc), that
Solution. Let . Choose . Since , this fraction is defined. Now, let . Then we have Since , we can multiply both sides of this inequality by . Since , we have , so . By previous proof, we have . Therefore, we can rewrite our inequality as Therefore, for any arbitrary , there exists an such that for all , . Thus, by definition, .
Exercise 3 If is a convergent for all , then . Similarly prove if is a convergent for all , then .
To gether these give a very useful way to bound a limit: if is a convergent sequence with for all , then .
Solution. Let and assume for contradiction . Then for some . Set . The interval () contains numbers strictly less then . By the definition of convergence, we know there exists an s.t. for all , lies in this interval. But this is a contradiction, because we know for all , so we know .
A similar argument works for the other part. Let . For the sake of contradiction, suppose . Then for some . Then, for some , we can let . Thus The interval () contains numbers strictly greater then . By the definition of convergence, we know there exists an s.t. for all , lies in this interval. But this is a contradiction, because we know for all , so we know .
Another way of expressing this idea:
Solution. First we consider the lower bound: assume for the sake of contradiction that but even though for all we know . Set : then we can find an where for all we know .
Unpacking this shows which implies (looking only at the upper bound, as that’s what’s relevant for us) Thus, must be less than the average of and , but since this average is less than ! This is a contradiction, since we know for all .
A very similar proof works for the upper bound: if and even though for all , setting allows us to get an where , whose lower bound unpacks into
Thus, must be greater than the average of and , which (since is greater than itself) must be greater than , contradicting the fact that .
Exercise 4 Let be two convergent sequences with the same limit. Prove directly (ie don’t just use the squeeze theorem) that the interleaved sequence defined by also converges, and has the same limit.
Solution. Call the common limit of and by . Choosing an we know that we can find an such that for all , and an such that $|y_n-L|<for all . Now let (the factor 4 is overkill and you can do better, but I’m not going to bother).
Choosing an arbitrary , we want to show that . Since is made from a mix of terms from and we have two cases. First consider if is even, so we can write , and as the terms are interleaved every other. Now, as we see implies that and so in particular, . Thus, by our assumption on , we have that , so as desired.
The reasoning is identical for odd, where now and . Since we know , thus so or equivalently .
Exercise 5 Use the squeeze theorem to prove that Hint: show this sequence is positive for all large enough , and then bound it above by an easy-to-compute sequence that we can show goes to zero. Notice how much easier this is than trying to find an for a given !
Solution. Starting with the given fraction, two things that we can do to make a larger sequence it is to make it a smaller denominator, and a larger numerator, And a way to make a smaller sequence is to use the sequence . This is because, after the fraction stays positive.
Now we just need to work with the upper bound a little more: since we see that and
That means we can replace our upper bound with this:
And since we have proven that and if we can apply this to and see it goes to zero.
So we can see that the limit must be , via the squeeze theorem.
Exercise 6 Prove that for every there exists a sequence of irrationals with .
Solution. Let be arbitrary. We know that the sequence has the property that . Similarly, we know that the sequence has the property that . So, by the sum law for limits, we know that lim = lim + lim = x + 0 = x. Let’s define the sequence as , so .
This means that there exists a sequence such that , but note that this doesn’t require that is solely irrational numbers. However, by the density of the irrationals in the real numbers (proved in an earlier homework), we know that there exists some irrational number in between any . So, lets set and . According to the density of the irrationals, there must be an irrational number such that for all . Lets form a sequence , where , dictated by the inequality in the previous sentence. This sequence is made of only irrational numbers, and satisfies the equation , so by the squeeze theorem, .
Exercise 7 In this problem you give an alternative proof that for all than the textbook’s (which used Bernoulli’s inequality). Instead, you’ll make use of the geometric sum from two assignments ago!
To recall, this stated that for any and ,
- First consider only the case that . Show that the geometric sum can be rewritten , and use this to prove that . Hint: apply it to and do some estimating.
- Use this to show that for all , and then prove (either directly, or using the squeeze theorem) that this implies .
- Now consider the case and show the same. Hint: if then , and perhaps you can do a similar trick to the textbook?
Solution. We begin with algebra: clearing denominators and multiplying the geometric sum from a previous homework by to get in the desired form. Substituting in to the geometric sum equality gives In the case , we know by previous homework that taking roots and powers preserves inequalities, so and so . Thus, as there are terms in the sum. This gives the inequality we were after:
Dividing this inequality by gives an upper bound for : . And recalling that we already know we see the left side of this inequality is positive so we’ve squeezed between two known quantities:
From here we prove directly that , though you may also choose to combine the squeeze theorem and limit laws for sums. Let and choose . Then for we are guaranteed
and so , meaning .
Next, we consider the case where . This means is greater than and so the above argument applies here:
Using our limit laws, we can find the limit of the reciprocal (since and the limit is nonzero)
But ! Thus
Exercise 8 (Limit of Products and Reciprocals) The textbook gives the scratch work of an argument for proving the following two claims:
- If are convergent sequences then converges, and
- If is a convergent sequence where and , then is convergent, with .
Take the scratch work for these two theorems and rewrite as rigorous proofs (starting by choosing an , then proposing and in terms of , and finally proving that the required inequality holds for all .)
Solution. This is just asking you to write rigorously (and in the right order) the sketch argument provided in the textbook, for practice.
Exercise 9 Use the limit laws to rigorously prove the following sequence converges, and to calculate its limit.
Hint: to apply the limit laws we need to work from the “inside out”: that is, before concluding the limit of a sum exists we need to first prove the limit of each of the terms does! Hint 2: think about the kind of “algebra simplifying” you do in Calc 1! Do this same algebra, to then rigorously apply limit laws together with the fact that
Solution. We start with the inside most fraction . Dividing the numerator and denominator both by yields the equivalent fraction To prove the numerator converges, we use the limit theorems for products and sums. Since we know converges with limit . Subtracting from the constant sequence , we see . The denominator is similar: since we see Since additionally the denominator is never zero (as so and ), the limit law for quotients applies to give
Now, as each term in this sequence of fractions is positive, we can apply the limit law for square roots of convergent sequences to conclude
Since we can apply our basic limit for |a|<1$ to see , and thus by the limit law for sums
Finally, we make one more applicaiton of the limit theorem for roots: since the limit just calculated is nonnegative, we have